diff --git a/CANADA_MO/download_script/download.py b/CANADA_MO/download_script/download.py new file mode 100644 index 0000000000000000000000000000000000000000..ce6dcd8e50bf69655d41024028d428edc68ed62c --- /dev/null +++ b/CANADA_MO/download_script/download.py @@ -0,0 +1,91 @@ +# ----------------------------------------------------------------------------- +# Author: Jiawei Liu +# Date: 2024-11-22 +# ----------------------------------------------------------------------------- +''' +Download script for CANADA MO + +Notes: +1. 1969-1993 no solution +2. 1994-1999 problem and solution are separated in two file +3. 2000+ problem and solution in the same file + +To run: +`python CANADA_MO\download_script\download.py` +''' + +import requests +from requests.adapters import HTTPAdapter +from tqdm import tqdm +from bs4 import BeautifulSoup +from urllib.parse import urljoin +from urllib3.util.retry import Retry +from pathlib import Path + + +def build_session( + max_retries: int = 3, + backoff_factor: int = 2, + session: requests.Session = None +) -> requests.Session: + """ + Build a requests session with retries + + Args: + max_retries (int, optional): Number of retries. Defaults to 3. + backoff_factor (int, optional): Backoff factor. Defaults to 2. + session (requests.Session, optional): Session object. Defaults to None. + """ + session = session or requests.Session() + adapter = HTTPAdapter(max_retries=Retry(total=max_retries, backoff_factor=backoff_factor)) + session.mount("http://", adapter) + session.mount("https://", adapter) + session.headers.update({ + "User-Agent": "Mozilla/5.0 (Windows NT 10.0; Win64; x64) AppleWebKit/537.36 (KHTML, like Gecko) Chrome/58.0.3029.110 Safari/537.3" + }) + + return session + + +def main(): + req_session = build_session() + base_url = "https://cms.math.ca/Competitions/CMO/" + + output_dir = Path(__file__).parent.parent / "raw" + output_dir.mkdir(parents=True, exist_ok=True) + + resp = req_session.get(base_url) + soup = BeautifulSoup(resp.text, "html.parser") + + cmo_year_a_ele = soup.find_all("a", href=lambda t: "#cmo" in t) + + for cy_a_ele in tqdm(cmo_year_a_ele): + year = cy_a_ele.get_text(strip=True) + + # Get problem and solution link elements + pro_link_ele = cy_a_ele.parent.parent.select_one("td:nth-child(2)").find("a") + sol_link_ele = cy_a_ele.parent.parent.select_one("td:nth-child(3)").find("a") + + donwload_urls = [] + if sol_link_ele: + donwload_urls.append(sol_link_ele["href"]) + else: + donwload_urls.append(pro_link_ele["href"]) + + # Year is between 1994 and 1999, download both problem and solution + if int(year) >= 1994 and int(year) <= 1999: + donwload_urls.append(pro_link_ele["href"]) + + for du in donwload_urls: + output_file = output_dir / f"en-{du.split('/')[-1]}" + + # Skip if file already exists + if output_file.exists(): + continue + + pdf_resp = req_session.get(du) + output_file.write_bytes(pdf_resp.content) + + +if __name__ == "__main__": + main() \ No newline at end of file diff --git a/CANADA_MO/md/en-2021CMO_solutions_en-1.md b/CANADA_MO/md/en-2021CMO_solutions_en-1.md new file mode 100644 index 0000000000000000000000000000000000000000..24ad31b52a617577ee81e6ad80bd3c98b1af3136 --- /dev/null +++ b/CANADA_MO/md/en-2021CMO_solutions_en-1.md @@ -0,0 +1,172 @@ +# Canadian Mathematical Olympiad 2021 + +## Official Solutions + +## A full list of our competition sponsors and partners is available online at https://cms.math.ca/competitions/competition-sponsors/ + +## Note: Each problem starts on a new page. + +Problem No. 1. + +Let $A B C D$ be a trapezoid with $A B$ parallel to $C D,|A B|>|C D|$, and equal edges $|A D|=|B C|$. Let $I$ be the center of the circle tangent to lines $A B, A C$ and $B D$, where $A$ and $I$ are on opposite sides of $B D$. Let $J$ be the center of the circle tangent to lines $C D, A C$ and $B D$, where $D$ and $J$ are on opposite sides of $A C$. Prove that $|I C|=|J B|$. + +Solution. Let $\{P\}=A C \cap B D$ and let $\angle A P B=180-2 a$. Since $A B C D$ is an isosceles trapezoid, $A P B$ is an isosceles triangle. Therefore $\angle P B A=a$, which implies that $\angle P B I=90^{\circ}-a / 2$ since $I$ lies on the external bisector of $\angle P B A$. Since $I$ lies on the bisector of $\angle C P B$, it follows that $\angle B P I=a$ and hence that $I P B$ is isosceles with $|I P|=|P B|$. Similarly $J P C$ is isosceles with $|J P|=|P C|$. So, in the triangles $C P I$ and $B P J$ we have $P I \equiv P B$ and $P J \equiv C P$. Since $I$ and $J$ both lie on the internal bisector of $\angle B P C$, it follows that triangles $C P I$ and $B P J$ are congruent. Therefore $|I C|=|J B|$. + +A competition of the Canadian Mathematical Society and supported by the Actuarial Profession. + + +Expertise. Insight. Solutions. + +## Canadian Mathematical Olympiad 2021 + +## Problem No. 2. + +Let $n \geq 2$ be some fixed positive integer and suppose that $a_{1}, a_{2}, \ldots, a_{n}$ are positive real numbers satisfying $a_{1}+a_{2}+\cdots+a_{n}=2^{n}-1$. + +Find the minimum possible value of + +$$ +\frac{a_{1}}{1}+\frac{a_{2}}{1+a_{1}}+\frac{a_{3}}{1+a_{1}+a_{2}}+\cdots+\frac{a_{n}}{1+a_{1}+a_{2}+\cdots+a_{n-1}} . +$$ + +Solution. We claim the the minimum possible value of this expression is $n$. Observe that by AM-GM, we have that + +$$ +\begin{aligned} +\frac{a_{1}}{1}+ & \frac{a_{2}}{1+a_{1}}+\cdots+\frac{a_{n}}{1+a_{1}+a_{2}+\cdots+a_{n-1}} \\ +& =\frac{1+a_{1}}{1}+\frac{1+a_{1}+a_{2}}{1+a_{1}}+\cdots+\frac{1+a_{1}+a_{2}+\cdots+a_{n}}{1+a_{1}+a_{2}+\cdots+a_{n-1}}-n \\ +& \geq n \cdot \sqrt[n]{\frac{1+a_{1}}{1} \cdot \frac{1+a_{1}+a_{2}}{1+a_{1}} \cdots \frac{1+a_{1}+a_{2}+\cdots+a_{n}}{1+a_{1}+a_{2}+\cdots+a_{n-1}}}-n \\ +& =n \cdot \sqrt[n]{1+a_{1}+a_{2}+\cdots+a_{n}}-n \\ +& =2 n-n=n . +\end{aligned} +$$ + +Furthermore, equality is achieved when $a_{k}=2^{k-1}$ for each $1 \leq k \leq n$. + +## Canadian Mathematical Olympiad 2021 + +## Problem No. 3. + +At a dinner party there are $N$ hosts and $N$ guests, seated around a circular table, where $N \geq 4$. A pair of two guests will chat with one another if either there is at most one person seated between them or if there are exactly two people between them, at least one of whom is a host. Prove that no matter how the 2 N people are seated at the dinner party, at least $N$ pairs of guests will chat with one another. + +Solution. Let a run refer to a maximal group of consecutive dinner party guests all of whom are the same type (host or guest). Suppose that there are exactly $k$ runs of hosts and $k$ runs of guests. Let $G_{i}$ and $H_{i}$ denote the number of runs of guests and hosts, respectively, of length exactly $i$. Furthermore, let $X$ denote the number of hosts surrounded by two runs of guests, both of length exactly 1 . We claim that the number of pairs of guests who chat is at least + +$$ +2 N-3 k+G_{1}+2 H_{1}+H_{2}-X +$$ + +The number of pairs of guests who chat with no host between them is at least the sum of $\max \{2 \ell-3,0\}$ over all guest run lengths $\ell$. This sum is at least $2 N-3 k+G_{1}$. The number of pairs of guests who chat with exactly two hosts between them is $H_{2}$. Furthermore, the number of pairs of guests who chat with exactly one host between them is at least $2 H_{1}-X$. This is because any host surrounded by two runs of guests causes at least two pairs of guests to chat unless these runs are both of length exactly 1 . This proves the claim. Now note that + +$$ +2 H_{1}+H_{2}+N \geq 3 k +$$ + +because each run of hosts contributes at least three to the left hand side. Furthermore, pairing each run counted in $X$ with the guest run of length 1 immediately following it in clockwise order shows that $G_{1} \geq X$. Combining these inequalities yields that $2 N-3 k+G_{1}+2 H_{1}+H_{2}-X \geq N$, completing the proof of the desired result. + +## Canadian Mathematical Olympiad 2021 + +## Problem No. 4. + +A function $f$ from the positive integers to the positive integers is called Canadian if it satisfies + +$$ +\operatorname{gcd}(f(f(x)), f(x+y))=\operatorname{gcd}(x, y) +$$ + +for all pairs of positive integers $x$ and $y$. +Find all positive integers $m$ such that $f(m)=m$ for all Canadian functions $f$. + +Solution. Define an $m \in \mathbb{N}$ to be good if $f(m)=m$ for all such $f$. It will be shown that $m$ is good if and only if $m$ has two or more distinct prime divisors. Let $P(x, y)$ denote the assertion + +$$ +\operatorname{gcd}(f(f(x)), f(x+y))=\operatorname{gcd}(x, y) +$$ + +for a pair $x, y \in \mathbb{N}$. Let $x$ be a positive integer with two or more distinct prime divisors and let $p^{k}$ be largest power of one of these prime divisors such that $p^{k} \mid x$. If $x=p^{k} \cdot q$, then $p^{k}$ and $q$ are relatively prime and $x>p^{k}, q>1$. By $P(q, x-q)$, + +$$ +\operatorname{gcd}(f(f(q)), f(x-q+q))=\operatorname{gcd}(f(f(q)), f(x))=\operatorname{gcd}(q, x-q)=q +$$ + +which implies that $q \mid f(x)$. By $P\left(p^{k}, x-p^{k}\right)$, + +$$ +\operatorname{gcd}\left(f\left(f\left(p^{k}\right)\right), f\left(x-p^{k}+p^{k}\right)\right)=\operatorname{gcd}\left(f\left(f\left(p^{k}\right)\right), f(x)\right)=\operatorname{gcd}\left(p^{k}, x-p^{k}\right)=p^{k} +$$ + +which implies that $p^{k} \mid f(x)$. Since $p^{k}$ and $q$ are relatively prime, $x=p^{k} \cdot q$ divides $f(x)$, which implies that $f(x) \geq x$. Now assume for contradiction that $f(x)>x$. Let $y=f(x)-x>0$ and note that, by $P(x, y)$, it follows that + +$$ +f(f(x))=\operatorname{gcd}(f(f(x)), f(x+f(x)-x))=\operatorname{gcd}(x, f(x)-x)=\operatorname{gcd}(x, f(x)) . +$$ + +Therefore $f(f(x)) \mid x$ and $f(f(x)) \mid f(x)$. By $P(x, x)$, it follows that + +$$ +\operatorname{gcd}(f(f(x)), f(2 x))=\operatorname{gcd}(x, x)=x +$$ + +This implies that $x \mid f(f(x))$, which when combined with the above result, yields that $f(f(x))=x$. Since $x \mid f(x)$ and $x$ is divisible by at least two distinct prime numbers, $f(x)$ is also divisible by at least two distinct prime numbers. As shown previously, this implies that $f(x) \mid f(f(x))=x$, which is a contradiction since $f(x)>x$. Therefore $f(x)=x$ for all positive integers $x$ with two or more distinct prime divisors. + +Now it will be shown that all $m \in \mathbb{N}$ such that either $m$ has one prime divisor or $m=1$ are not good. In either case, let $m=p^{k}$ where $k \geq 0$ and $p$ is a prime number and consider the function satisfying that $f\left(p^{k}\right)=p^{k+1}, f\left(p^{k+1}\right)=p^{k}$ and $f(x)=x$ for all $x \neq p^{k}, p^{k+1}$. Note that this function also satisfies that $f(f(x))=x$ for all positive integers $x$. If $x+y \neq p^{k}, p^{k+1}$, then $P(x, y)$ holds by the Euclidean + +## Canadian Mathematical Olympiad 2021 + +algorithm since $f\left(f((x))=x\right.$ and $f(x+y)=x+y$. If $x+y=p^{k+1}$, then $P(x, y)$ is equivalent to $\operatorname{gcd}\left(x, p^{k}\right)=\operatorname{gcd}\left(x, p^{k+1}-x\right)=\operatorname{gcd}\left(x, p^{k+1}\right)$ for all $x
z$, as desired.
+
+Case 2. At least two carries: $x+y$ and $x+z$ carry and $d_{\ell+1}(y)=d_{\ell+1}(z)=0$, or cyclic equivalent. ( $y+z$ may or may not carry.)
+
+Let $i$ be maximal such that $d_{\ell+1}(x)=\cdots=d_{\ell+i}(x)=1$ (possibly $i=0$ ). By maximality of $\ell, d_{\ell+1}(y)=$ $\cdots=d_{\ell+i}(y)=d_{\ell+1}(z)=\cdots=d_{\ell+i}(z)=0$. By maximality of $i, d_{\ell+i+1}(x)=0$.
+
+If $d_{\ell+i+1}(y)=d_{\ell+i+1}(z)=0$, then $d_{\ell+i+1}(x+y)=d_{\ell+i+1}(x+z)=1$, so $d_{\ell+i+1}\left(x^{\prime}\right)=1$. The binary representations of $x$ and $x^{\prime}$ agree to the left of the $2^{\ell+i+1}$ position, so $x^{\prime}>x$.
+
+## Canadian Mathematical Olympiad 2021
+
+Otherwise, WLOG $d_{\ell+i+1}(y)=1$ and $d_{\ell+i+1}(z)=0$. (Note that, here we in fact have $i \geq 1$.) Then $d_{\ell+i+1}(y+z)=d_{\ell+i+1}(x+z)=1$, so $d_{\ell+i+1}\left(z^{\prime}\right)=1$. The binary representations of $z$ and $z^{\prime}$ agree to the left of the $2^{\ell+i+1}$ position, so $z^{\prime}>z$.
+
+Case 3. At least two carries, and the condition in Case 2 does not occur.
+WLOG let $x+y, x+z$ involve carries. Since the condition in Case 2 does not occur, $d_{\ell+1}(y)=1$ or $d_{\ell+1}(z)=1$. In either case, $d_{\ell+1}(x)=0$. WLOG $d_{\ell+1}(y)=1$ and $d_{\ell+1}(z)=0$.
+Since the condition in Case 2 does not occur, $y+z$ does not involve a carry from the $2^{\ell}$ position. (Otherwise, $x+y$ and $y+z$ carry and $d_{\ell+1}(x)=d_{\ell+1}(z)=0$.) Then $d_{\ell+1}(x+z)=d_{\ell+1}(y+z)=1$, so $d_{\ell+1}\left(z^{\prime}\right)=1$. The binary representations of $z$ and $z^{\prime}$ agree to the left of the $2^{\ell+1}$ position, so $z^{\prime}>z$.
+
+Proof of Claim 1. Proceed by strong induction on $x+y+z$. There is no base case.
+Suppose by induction the claim holds for all $(x, y, z)$ with sum less than $N$. Consider a triple $(x, y, z)$ with $x+y+z=N$.
+Suppose no two of $x, y, z$ overlap. If all moves from this position lead to positions with a negative coordinate, $(x, y, z)$ is a losing position, as claimed. Otherwise, the player increases or decreases all coordinates by $k$. Consider the smallest $m$ such that $d_{m}(k)=1$. The player's move will toggle each of $d_{m}(x), d_{m}(y), d_{m}(z)$. Since at most one of the original $d_{m}(x), d_{m}(y), d_{m}(z)$ is 1 , at least two of the new $d_{m}(x), d_{m}(y), d_{m}(z)$ will be 1 . So, two of the new $x, y, z$ overlap. By induction, the new $(x, y, z)$ is winning. Thus the original $(x, y, z)$ is losing, as claimed.
+
+Conversely, suppose at least one pair of $x, y, z$ overlap. By Lemma 1, at least one of $x 2024$. Then note that
+
+$$
+v_{p}(1!\cdot 2!\cdots 2024!)=(2024-p+1)+(2024-2 p+1)+\ldots+(2024-k p+1)
+$$
+
+In particular, let $p$ be in $\left(\frac{2024}{4}, \frac{2024}{2}\right)$. By Bertrand's Postulate, such a prime $p$ exists (and $p$ must also be odd). Further, the corresponding $k$ is either 2 or 3 . Either way, $v_{p}(1!\cdot 2!\cdots 2024$ !) is odd from the above formula, and so $1!\cdot 2!\cdots 2024$ ! cannot be a perfect square.
+
+Solution 2. As in the first solution, we prove $1!\cdot 2!\cdots 2024$ ! is not a perfect square. To do this, note that we can rewrite the product as $(1!)^{2} \cdot 2 \cdot(3!)^{2} \cdot 4 \cdots(2023!)^{2} \cdot 2024$ which is
+
+$$
+2 \cdot 4 \cdots 2024 \cdot(1!\cdot 3!\cdots 2023!)^{2}=1012!\cdot\left(2^{1012} \cdot 1!\cdot 3!\cdots 2023!\right)^{2}
+$$
+
+so it is sufficient to verify 1012 ! is not a perfect square. This can be verified by either noticing the prime 1009 only appears as a factor of 1012 ! once, or by evaluating $v_{2}(1012!)=1005$.
+
+P3. Let $N$ be the number of positive integers with 10 digits $\overline{d_{9} d_{8} \cdots d_{1} d_{0}}$ in base 10 (where $0 \leq d_{i} \leq 9$ for all $i$ and $d_{9}>0$ ) such that the polynomial
+
+$$
+d_{9} x^{9}+d_{8} x^{8}+\cdots+d_{1} x+d_{0}
+$$
+
+is irreducible in $\mathbb{Q}$. Prove that $N$ is even.
+(A polynomial is irreducible in $\mathbb{Q}$ if it cannot be factored into two non-constant polynomials with rational coefficients.)
+
+Solution. Let $f(x)=d_{9} x^{9}+d_{8} x^{8}+\cdots+d_{1} x+d_{0}$. If $d_{0}=0$, then $f(x)$ is divisible by $x$ and thus reducible, so we may ignore all such polynomials. The remaining polynomials all have nonzero leading and constant coefficients.
+
+For any polynomial $p(x)$ of degree $n$ with nonzero leading and constant coefficients, say $p(x)=a_{n} x^{n}+a_{n-1} x^{n-1}+\cdots+a_{1} x+a_{0}$, define $\bar{p}(x)$ to be the reversed polynomial $a_{0} x^{n}+$ $a_{1} x^{n-1}+\cdots+a_{n-1} x+a_{n}$. Observe that $\bar{p}(x)$ also has degree $n$ and furthermore, $\bar{p}(x)=$ $x^{n}\left(a_{0}+a_{1}\left(\frac{1}{x}\right)+\cdots+a_{n-1}\left(\frac{1}{x}\right)^{n-1}+a_{n}\left(\frac{1}{x}\right)^{n}\right)=x^{n} p\left(\frac{1}{x}\right)$.
+Consider pairing each $f(x)$ with $\bar{f}(x)$ whenever $f(x) \neq \bar{f}(x)$. If $f(x)$ is reducible, it can be factored as $f(x)=g(x) h(x)$ where $\operatorname{deg} g, \operatorname{deg} h \geq 1$. Because the leading and constant coefficients of $f(x)$ are nonzero, so are the leading and constant coefficients of $g(x)$ and $h(x)$. Hence $\bar{g}(x)$ and $\bar{h}(x)$ are well defined with $\operatorname{deg} \bar{g}=\operatorname{deg} g \geq 1$ and $\operatorname{deg} \bar{h}=\operatorname{deg} h \geq 1$. Furthermore,
+
+$$
+\bar{f}(x)=x^{9} f\left(\frac{1}{x}\right)=x^{9} g\left(\frac{1}{x}\right) h\left(\frac{1}{x}\right)=\left(x^{\operatorname{deg} g} g\left(\frac{1}{x}\right)\right)\left(x^{\operatorname{deg} h} h\left(\frac{1}{x}\right)\right)=\bar{g}(x) \bar{h}(x)
+$$
+
+Thus $\bar{f}(x)=\bar{g}(x) \bar{h}(x)$ is a factorization of $\bar{f}(x)$ into two non-constant polynomials, so $\bar{f}(x)$ is also reducible. Therefore $f(x)$ is irreducible if and only if $\bar{f}(x)$ is irreducible, so considering each pair, there are an even number of irreducible polynomials with $f(x) \neq \bar{f}(x)$.
+Finally, note that if $f(x)=\bar{f}(x)$, then $d_{i}=d_{9-i}$ for each $i$. In such a case, we have $f(-1)=$ $\left(d_{0}-d_{9}\right)+\left(d_{2}-d_{7}\right)+\left(d_{4}-d_{5}\right)+\left(d_{6}-d_{3}\right)+\left(d_{8}-d_{1}\right)=0$, so by the Factor Theorem, $(x+1)$ is a factor of $f(x)$. Therefore these remaining polynomials are all reducible.
+
+P4. Centuries ago, the pirate Captain Blackboard buried a vast amount of treasure in a single cell of an $M \times N(2 \leq M, N)$ grid-structured island. You and your crew have reached the island and have brought special treasure detectors to find the cell with the treasure. For each detector, you can set it up to scan a specific subgrid $[a, b] \times[c, d]$ with $1 \leq a \leq b \leq M$ and $1 \leq c \leq d \leq N$. Running the detector will tell you whether the treasure is in the region or not, though it cannot say where in the region the treasure was detected. You plan on setting up $Q$ detectors, which may only be run simultaneously after all $Q$ detectors are ready. In terms of $M$ and $N$, what is the minimum $Q$ required to guarantee your crew can determine the location of Blackboard's legendary treasure?
+
+Solution 1. Let $m=\left\lceil\frac{M}{2}\right\rceil$ and $n=\left\lceil\frac{N}{2}\right\rceil$. We claim that the minimal $Q$ is $m+n$. For the construction, start with $m$ detectors covering $[i, i+m-1] \times[1, N]$ for $1 \leq i \leq m$. For every pair of rows, there is a detector that covers one row but not the other, hence this determines the row of the treasure. Similarly, placing $n$ detectors covering $[1, M] \times[i, i+n-1]$ for $1 \leq i \leq n$ determines the column, and thus the location of the treasure.
+
+For the bound, we require the following lemma.
+Lemma. $A 1 \times k$ island requires at least $\left\lceil\frac{k}{2}\right\rceil$ detectors.
+Proof. Consider the $k-1$ lines separating the cells. If one of these lines is not covered by any detector, then these cells are indistinguishable. Similarly, if neither of the vertical lines at the ends are covered, then the first and last cells are indistinguishable. In particular, at least $k$ vertical lines need to be covered by the detectors. A detector covers 2 vertical lines, giving the result.
+
+In general, consider the first row. Since the cells are distinguishable, by the lemma there must be at least $n$ detectors that intersect it non-trivially (as in, cover between 1 and $N-1$ of the cells). The analogous result holds for the last row and the first/last columns, giving $2 m+2 n$ detectors, where a detector may be counted multiple times.
+
+If a detector intersected at least three of these sets, say it intersected the first row and the first and last columns. Therefore it covers the entire width of the island, and does not actually distinguish any cells in the first row, contradiction.
+
+Therefore each detector contributes to at most 2 of the above $2 m+2 n$ detectors, giving the final lower bound of $\frac{2 m+2 n}{2}=m+n$ detectors required, as desired.
+
+Solution 2. The following alternative approach from CMO competitor Marvin Mao of Bergen County Academies is another full solution.
+
+Take the same construction as in Solution 1. For the bound, consider the following sets:
+
+- $S_{\mathrm{CR}}:=\{\{(1,1),(1, N)\},\{(M, 1),(M, N)\}\}$, i.e. the pairs of corners on the same row;
+- $S_{\mathrm{CC}}:=\{\{(1,1),(M, 1)\},\{(1, N),(M, N)\}\}$, i.e. the pairs of corners on the same column;
+- $S_{\mathrm{R}}:=\{\{(x, i),(x, i+1)\}: x \in\{1, M\}, 1 \leq i \leq N-1\}$, i.e. the pairs of adjacent edges on the first/last row;
+- $S_{\mathrm{C}}:=\{\{(i, x),(i+1, x)\}: 1 \leq i \leq M-1, x \in\{1, N\}$, $\}$, i.e. the pairs of adjacent edges on the first/last column.
+
+For each detector, we assign it a score $\left(x_{\mathrm{CR}}, x_{\mathrm{CC}}, x_{\mathrm{R}}, x_{\mathrm{C}}\right)$, where $x_{\mathrm{i}}$ is the number of pairs of cells in $S_{\mathrm{i}}$ for which the detector covers exactly one of the two cells. The possible scores of the detectors are as follows:
+
+| What the detector hits | Score |
+| :---: | :---: |
+| No edges | $(0,0,0,0)$ |
+| One edge, no corners | $(0,0,2,0)$ or $(0,0,0,2)$ |
+| Two edges, no corners | $(0,0,4,0)$ or $(0,0,0,4)$ |
+| One corner | $(1,1,1,1)$ |
+| Two corners | $(2,0,2,0)$ or $(0,2,0,2)$ |
+| $>2$ corners or edges | $(0,0,0,0)$ |
+
+In order to determine the treasure, the total component-wise sum of scores of the detectors needs to be at least $(2,2,2 N-2,2 M-2)$, since we need to tell apart each of the pairs of cells. The sum of these components is $2 M+2 N$, and based on the analysis above, each detector adds a total component sum of at most 4 , giving at least $\left\lceil\frac{2 M+2 N}{4}\right\rceil=\left\lceil\frac{M+N}{2}\right\rceil$ detectors.
+
+This is equal to $\left\lceil\frac{M}{2}\right\rceil+\left\lceil\frac{N}{2}\right\rceil$ except if both $M, N$ are odd. In this case, if there is at least one more detector, then we have the required bound, so assume otherwise. In particular, we must achieve exactly the score $(2,2,2 N-2,2 M-2)$, with each detector contributing 4 to the total component sum.
+
+In particular, to fill out the first two components, we must either have two detectors scoring $(1,1,1,1)$, or two detectors scoring $(2,0,2,0)$ and $(0,2,0,2)$. This yields a total score of $(2,2,2,2)$, leaving us with achieving exactly $(0,0,2 N-4,2 M-4)$ from the rest. Since we cannot have a non-zero score in the first two entries and must have a total component sum of 4 , we can only use detectors scoring $(0,0,4,0)$ or $(0,0,0,4)$. But $2 N-4,2 M-4 \equiv 2$ $(\bmod 4)$, which is a contradiction.
+Therefore all situations require at least $\left\lceil\frac{M}{2}\right\rceil+\left\lceil\frac{N}{2}\right\rceil$ detectors.
+
+P5. Initially, three non-collinear points, $A, B$, and $C$, are marked on the plane. You have a pencil and a double-edged ruler of width 1. Using them, you may perform the following operations:
+
+- Mark an arbitrary point in the plane.
+- Mark an arbitrary point on an already drawn line.
+- If two points $P_{1}$ and $P_{2}$ are marked, draw the line connecting $P_{1}$ and $P_{2}$.
+- If two non-parallel lines $\ell_{1}$ and $\ell_{2}$ are drawn, mark the intersection of $\ell_{1}$ and $\ell_{2}$.
+- If a line $\ell$ is drawn, draw a line parallel to $\ell$ that is at distance 1 away from $\ell$ (note that two such lines may be drawn).
+
+Prove that it is possible to mark the orthocenter of $A B C$ using these operations.
+
+## Solution 1.
+
+Claim 1. It is possible to draw internal/external angle bisectors.
+Proof. Let $A, B, C$ be marked. To bisect $\angle A B C$, draw the parallel line to $A B$ unit 1 away from it on the opposite side as $C$, and draw the parallel line to $B C$ unit 1 away from it on the opposite side as $A$. Let these lines intersect at $D$. Then $B D$ is the internal angle bisector of $\angle A B C$. We can construct external angle bisectors similarly by drawing the line on the same side as $A$ for the second line instead.
+
+Corollary 2. It is possible to mark the incenters and excenters of a triangle $A B C$.
+Proof. Draw in the internal/external bisectors of all three angles and intersect them.
+Claim 3. It is possible to mark the midpoint of any segment $A B$.
+Proof. Let $B$ and $C$ be marked. Draw an arbitrary point $A$ not on line $B C$. Draw a line parallel to $B C$ unit 1 away from it on the opposite side as $A$, and let this line intersect $A B$ at $D$ and $A C$ at $E$. Let $B E$ and $C D$ intersect at $F$, and let $A F$ intersect $B C$ at $M$. Then by Ceva's Theorem, $M$ is the midpoint of $B C$.
+
+Corollary 4. It is possible to mark the centroid of $A B C$.
+Proof. Draw the midpoint $D$ of $B C$ and the midpoint $E$ of $A C$, and intersect $A D$ with BE.
+
+Claim 5. It is possible to draw the perpendicular bisector of any segment BC.
+
+Proof. Let $B$ and $C$ be marked. Draw an arbitrary point $A$ not on line $B C$. Construct the incenter $I$ and $A$-excenter $I_{A}$ of $A B C$. Draw the midpoint $M$ of $B C$ and midpoint $N$ of $I I_{A}$. By the incenter-excenter lemma, $N$ is the midpoint of the $\operatorname{arc} \overparen{B C}$ not containing $A$, so $M N$ is the perpendicular bisector of $B C$.
+
+Corollary 6. It is possible to mark the circumcenter of $A B C$.
+Proof. Draw and intersect the perpendicular bisectors of $B C$ and $A C$.
+
+Claim 7. Given two marked points $A$ and $B$, it is possible to mark the point $C$ such that $\overrightarrow{B C}=\frac{1}{2} \overrightarrow{A B}$.
+
+Proof. Draw an arbitrary point $D$ not on line $A B$. Draw the midpoint $M$ of $A D$. Draw the midpoint $M_{1}$ of $B D$ and the midpoint $M_{2}$ of $M D$, and let $M_{1} M_{2}$ intersect $A B$ at $C$. Then $M_{1} M_{2} \| B M$ and $M M_{2}=\frac{1}{2} M D=\frac{1}{2} A M$, so $B C=\frac{1}{2} A B$.
+
+Claim 8. Given two marked points $A$ and $B$ and any positive real number $k$ such that $2 k \in \mathbb{Z}$, it is possible to mark the point $C$ such that $\overrightarrow{B C}=k \overrightarrow{A B}$.
+
+Proof. Note that by applying Claim 7 and marking the midpoint of $A B$, we can translate both $A$ and $B$ by $\frac{1}{2} \overrightarrow{A B}$. The claim now follows by applying this operation repeatedly.
+
+To finish, take the given triangle $A B C$ and mark its circumcenter $O$ and centroid $G$. Note that its orthocenter $H$ satisfies that $\overrightarrow{G H}=2 \overrightarrow{O G}$, so applying Claim 8 to $k=2$ now finishes the problem.
+
+Solution 2. Ming Yang of Brophy College Preparatory submitted the following short, elegant solution which also creates tools that are able to extend beyond the problem. This solution has been designated by the CMO as the Best Solution for 2024 and earns Ming Yang the Matthew Brennan Award this year.
+
+Start with Claims 1-3 of solution 1, allowing us to draw internal/external angle bisectors, in/excentres, and midpoints. We add one more claim.
+
+Claim 9. Given a point $P$ and a line $\ell_{1}$, it is possible to draw a line through $P$ parallel to $\ell_{1}$.
+
+Proof. Draw the line $\ell_{2}$ on the opposite side of $\ell_{1}$ to $P$, a distance 1 away. Draw arbitrary lines $P A B$ and $P C D$ with $A, C \in \ell_{1}, B, D \in \ell_{2}$. Let $E$ be the midpoint of $A C$, let $F=P E \cap \ell_{2}$, and let $Q=B E \cap F C$.
+
+Since $\triangle Q E C \sim \triangle Q B F$ and $\triangle P A E \sim \triangle P B F$, we have
+
+$$
+\frac{Q E}{Q B}=\frac{E C}{B F}=\frac{A E}{B F}=\frac{P A}{P B}
+$$
+
+so $\triangle B A E \sim \triangle B P Q$. In particular, $P Q$ is parallel to $A E$, as desired.
+In triangle $\triangle A B C$, draw the incentre $I$ and $A$-excentre $I_{A}$. Draw the midpoints $D$ of $B C$ and $M$ of $I I_{A}$. By the incentre-excentre lemma, $M$ is on the perpendicular bisector of $B C$, so $M D$ is perpendicular to $B C$. Finally, using Claim 9, we can draw a line through $A$ that is perpendicular to $B C$. Repeat this for $B$ and $A C$, and their intersection is the orthocentre of $\triangle A B C$, as required.
+
diff --git a/CANADA_MO/md/en-exam1969.md b/CANADA_MO/md/en-exam1969.md
new file mode 100644
index 0000000000000000000000000000000000000000..6558342a4ede3b012c2316125991fdadec990f75
--- /dev/null
+++ b/CANADA_MO/md/en-exam1969.md
@@ -0,0 +1,61 @@
+# Canadian Mathematical Olympiad
+
+## Problem 1
+
+Show that if $a_{1} / b_{1}=a_{2} / b_{2}=a_{3} / b_{3}$ and $p_{1}, p_{2}, p_{3}$ are not all zero, then
+
+$$
+\left(\frac{a_{1}}{b_{1}}\right)^{n}=\frac{p_{1} a_{1}^{n}+p_{2} a_{2}^{n}+p_{3} a_{3}^{n}}{p_{1} b_{1}^{n}+p_{2} b_{2}^{n}+p_{3} b_{3}^{n}}
+$$
+
+for every positive integer $n$.
+
+## PROBLEM 2
+
+Determine which of the two numbers $\sqrt{c+1}-\sqrt{c}, \sqrt{c}-\sqrt{c-1}$ is greater for any $c \geq 1$.
+
+## Problem 3
+
+Let $c$ be the length of the hypotenuse of a right angle triangle whose other two sides have lengths $a$ and $b$. Prove that $a+b \leq \sqrt{2} c$. When does the equality hold?
+
+Problem 4
+Let $A B C$ be an equilateral triangle, and $P$ be an arbitrary point within the triangle. Perpendiculars $P D, P E, P F$ are drawn to the three sides of the triangle. Show that, no matter where $P$ is chosen,
+
+$$
+\frac{P D+P E+P F}{A B+B C+C A}=\frac{1}{2 \sqrt{3}}
+$$
+
+## PROBLEM 5
+
+Let $A B C$ be a triangle with sides of lengths $a, b$ and $c$. Let the bisector of the angle $C$ cut $A B$ in $D$. Prove that the length of $C D$ is
+
+$$
+\frac{2 a b \cos \frac{C}{2}}{a+b}
+$$
+
+## PROBLEM 6
+
+Find the sum of $1 \cdot 1!+2 \cdot 2!+3 \cdot 3!+\cdots+(n-1)(n-1)!+n \cdot n!$, where $n!=$ $n(n-1)(n-2) \cdots 2 \cdot 1$.
+
+PROBLEM 7
+Show that there are no integers $a, b, c$ for which $a^{2}+b^{2}-8 c=6$.
+
+## Problem 8
+
+Let $f$ be a function with the following properties:
+
+1) $f(n)$ is defined for every positive integer $n$;
+2) $f(n)$ is an integer;
+3) $f(2)=2$;
+4) $f(m n)=f(m) f(n)$ for all $m$ and $n$;
+5) $f(m)>f(n)$ whenever $m>n$.
+
+Prove that $f(n)=n$.
+PROBLEM 9
+Show that for any quadrilateral inscribed in a circle of radius 1 , the length of the shortest side is less than or equal to $\sqrt{2}$.
+
+## PRoblem 10
+
+Let $A B C$ be the right-angled isosceles triangle whose equal sides have length $1 . P$ is a point on the hypotenuse, and the feet of the perpendiculars from $P$ to the other sides are $Q$ and $R$. Consider the areas of the triangles $A P Q$ and $P B R$, and the area of the rectangle $Q C R P$. Prove that regardless of how $P$ is chosen, the largest of these three areas is at least $2 / 9$.
+
+
diff --git a/CANADA_MO/md/en-exam1970.md b/CANADA_MO/md/en-exam1970.md
new file mode 100644
index 0000000000000000000000000000000000000000..0ba4cc635be808c78db2ff7563dfb5e469d9cb1f
--- /dev/null
+++ b/CANADA_MO/md/en-exam1970.md
@@ -0,0 +1,67 @@
+# Canadian Mathematical Olympiad
+
+1970
+
+## Problem 1
+
+Find all number triples $(x, y, z)$ such that when any one of these numbers is added to the product of the other two, the result is 2 .
+
+## Problem 2
+
+Given a triangle $A B C$ with angle $A$ obtuse and with altitudes of length $h$ and $k$ as shown in the diagram, prove that $a+h \geq b+k$. Find under what conditions $a+h=b+k$.
+
+## Problem 3
+
+A set of balls is given. Each ball is coloured red or blue, and there is at least one of each colour. Each ball weighs either 1 pound or 2 pounds, and there is at least one of each weight. Prove that there are 2 balls having different weights and different colours.
+
+## PROBLEM 4
+
+a) Find all positive integers with initial digit 6 such that the integer formed by deleting this 6 is $1 / 25$ of the original integer.
+b) Show that there is no integer such that deletion of the first digit produces a result which is $1 / 35$ of the original integer.
+
+## Problem 5
+
+A quadrilateral has one vertex on each side of a square of side-length 1. Show that the lengths $a, b, c$ and $d$ of the sides of the quadrilateral satisfy the inequalities
+
+$$
+2 \leq a^{2}+b^{2}+c^{2}+d^{2} \leq 4
+$$
+
+## Problem 6
+
+Given three non-collinear points $A, B, C$, construct a circle with centre $C$ such that the tangents from $A$ and $B$ to the circle are parallel.
+
+## PROBLEM 7
+
+Show that from any five integers, not necessarily distinct, one can always choose three of these integers whose sum is divisible by 3.
+
+## Problem 8
+
+Consider all line segments of length 4 with one end-point on the line $y=x$ and the other end-point on the line $y=2 x$. Find the equation of the locus of the midpoints of these line segments.
+
+## PROblem 9
+
+Let $f(n)$ be the sum of the first $n$ terms of the sequence
+
+$$
+0,1,1,2,2,3,3,4,4,5,5,6,6, \ldots
+$$
+
+a) Give a formula for $f(n)$.
+b) Prove that $f(s+t)-f(s-t)=s t$ where $s$ and $t$ are positive integers and $s>t$.
+
+Problem 10
+Given the polynomial
+
+$$
+f(x)=x^{n}+a_{1} x^{n-1}+a_{2} x^{n-2}+\cdots+a_{n-1} x+a_{n}
+$$
+
+with integral coefficients $a_{1}, a_{2}, \ldots, a_{n}$, and given also that there exist four distinct integers $a, b, c$ and $d$ such that
+
+$$
+f(a)=f(b)=f(c)=f(d)=5
+$$
+
+show that there is no integer $k$ such that $f(k)=8$.
+
diff --git a/CANADA_MO/md/en-exam1971.md b/CANADA_MO/md/en-exam1971.md
new file mode 100644
index 0000000000000000000000000000000000000000..d035d0e17b8856c43405132ae05b4187fe691b5e
--- /dev/null
+++ b/CANADA_MO/md/en-exam1971.md
@@ -0,0 +1,55 @@
+# Canadian Mathematical Olympiad
+
+## Problem 1
+
+$D E B$ is a chord of a circle such that $D E=3$ and $E B=5$. Let $O$ be the centre of the circle. Join $O E$ and extend $O E$ to cut the circle at $C$. (See diagram). Given $E C=1$, find the radius of the circle.
+
+## PROBLEM 2
+
+
+
+Let $x$ and $y$ be positive real numbers such that $x+y=1$. Show that
+
+$$
+\left(1+\frac{1}{x}\right)\left(1+\frac{1}{y}\right) \geq 9
+$$
+
+## PROblem 3
+
+$A B C D$ is a quadrilateral with $A D=B C$. If $\angle A D C$ is greater than $\angle B C D$, prove that $A C>B D$.
+
+## PROBLEM 4
+
+Determine all real numbers $a$ such that the two polynomials $x^{2}+a x+1$ and $x^{2}+x+a$ have at least one root in common.
+
+## Problem 5
+
+Let
+
+$$
+p(x)=a_{0} x^{n}+a_{1} x^{n-1}+\cdots+a_{n-1} x+a_{n}
+$$
+
+where the coefficients $a_{i}$ are integers. If $p(0)$ and $p(1)$ are both odd, show that $p(x)$ has no integral roots.
+
+## Problem 6
+
+Show that, for all integers $n, n^{2}+2 n+12$ is not a multiple of 121 .
+
+## PROBLEM 7
+
+Let $n$ be a five digit number (whose first digit is non-zero) and let $m$ be the four digit number formed from $n$ by deleting its middle digit. Determine all $n$ such that $n / m$ is an integer.
+
+## Problem 8
+
+A regular pentagon is inscribed in a circle of radius $r . P$ is any point inside the pentagon. Perpendiculars are dropped from $P$ to the sides, or the sides produced, of the pentagon.
+a) Prove that the sum of the lengths of these perpendiculars is constant.
+b) Express this constant in terms of the radius $r$.
+
+Problem 9
+Two flag poles of heights $h$ and $k$ are situated $2 a$ units apart on a level surface. Find the set of all points on the surface which are so situated that the angles of elevation of the tops of the poles are equal.
+
+## PROBLEM 10
+
+Suppose that $n$ people each know exactly one piece of information, and all $n$ pieces are different. Every time person A phones person B, A tells B everything that A knows, while B tells A nothing. What is the minimum number of phone calls between pairs of people needed for everyone to know everything? Prove your answer is a minimum.
+
diff --git a/CANADA_MO/md/en-exam1972.md b/CANADA_MO/md/en-exam1972.md
new file mode 100644
index 0000000000000000000000000000000000000000..eb33198e83cabce46e1f40434c5b8fc3fb71fcd6
--- /dev/null
+++ b/CANADA_MO/md/en-exam1972.md
@@ -0,0 +1,54 @@
+# Canadian Mathematical Olympiad
+
+1972
+
+## Problem 1
+
+Given three distinct unit circles, each of which is tangent to the other two, find the radii of the circles which are tangent to all three circles.
+
+## PROblem 2
+
+Let $a_{1}, a_{2}, \ldots, a_{n}$ be non-negative real numbers. Define $M$ to be the sum of all products of pairs $a_{i} a_{j}(i SOLUTIONS
+
+The solutions to the problems of the 1998 CMO presented below are taken from students papers.
+
+Some minor editing has been done - unnecesary steps have been eliminated and some wording has been changed to make the proofs clearer. But for the most part, the proofs are as submitted.
+
+## Solution to Problem 1 - David Arthur, Upper Canada College, Toronto, ON
+
+Let $a=30 k+r$, where $k$ is an integer and $r$ is a real number between 0 and 29 inclusive.
+
+Then $\left[\frac{1}{2} a\right]=\left[\frac{1}{2}(30 k+r)\right]=15 k+\left[\frac{r}{2}\right]$. Similarly $\left[\frac{1}{3} a\right]=10 k+\left[\frac{r}{3}\right]$ and $\left[\frac{1}{5} a\right]=6 k+\left[\frac{r}{5}\right]$.
+
+Now, $\left[\frac{1}{2} a\right]+\left[\frac{1}{3} a\right]+\left[\frac{1}{5} a\right]=a$, so $\left(15 k+\left[\frac{r}{2}\right]\right)+\left(10 k+\left[\frac{r}{3}\right]\right)+\left(6 k+\left[\frac{r}{5}\right]\right)=30 k+r$ and hence $k=r-\left[\frac{r}{2}\right]-\left[\frac{r}{3}\right]-\left[\frac{r}{5}\right]$.
+
+Clearly, $r$ has to be an integer, or $r-\left[\frac{r}{2}\right]-\left[\frac{r}{3}\right]-\left[\frac{r}{5}\right]$ will not be an integer, and therefore, cannot equal $k$.
+
+On the other hand, if $r$ is an integer, then $r-\left[\frac{r}{2}\right]-\left[\frac{r}{3}\right]-\left[\frac{r}{5}\right]$ will also be an integer, giving exactly one solution for $k$.
+
+For each $r(0 \leq r \leq 29), a=30 k+r$ will have a different remainder mod 30 , so no two different values of $r$ give the same result for $a$.
+
+Since there are 30 possible values for $r(0,1,2, \ldots, 29)$, there are then 30 solutions for $a$.
+
+Solution to Problem 2 - Jimmy Chui, Earl Haig S.S., North York, ON
+
+Since $\left(x-\frac{1}{x}\right)^{1 / 2} \geq 0$ and $\left(1-\frac{1}{x}\right)^{1 / 2} \geq 0$, then $0 \leq\left(x-\frac{1}{x}\right)^{1 / 2}+\left(1-\frac{1}{x}\right)^{1 / 2}=x$.
+
+Note that $x \neq 0$. Else, $\frac{1}{x}$ would not be defined so $x>0$.
+
+Squaring both sides gives,
+
+$$
+\begin{aligned}
+x^{2} & =\left(x-\frac{1}{x}\right)+\left(1-\frac{1}{x}\right)+2 \sqrt{\left(x-\frac{1}{x}\right)\left(1-\frac{1}{x}\right)} \\
+x^{2} & =x+1-\frac{2}{x}+2 \sqrt{x-1-\frac{1}{x}+\frac{1}{x^{2}}} .
+\end{aligned}
+$$
+
+Multiplying both sides by $x$ and rearranging, we get
+
+$$
+\begin{aligned}
+x^{3}-x^{2}-x+2 & =2 \sqrt{x^{3}-x^{2}-x+1} \\
+\left(x^{3}-x^{2}-x+1\right)-2 \sqrt{x^{3}-x^{2}-x+1}+1 & =0 \\
+\left(\sqrt{x^{3}-x^{2}-x+1}-1\right)^{2} & =0 \\
+\sqrt{x^{3}-x^{2}-x+1} & =1 \\
+x^{3}-x^{2}-x+1 & =1 \\
+x\left(x^{2}-x-1\right) & =0 \\
+x^{2}-x-1 & =0 \quad \text { since } x \neq 0 .
+\end{aligned}
+$$
+
+Thus $x=\frac{1 \pm \sqrt{5}}{2}$. We must check to see if these are indeed solutions.
+
+Let $\alpha=\frac{1+\sqrt{5}}{2}, \beta=\frac{1-\sqrt{5}}{2}$. Note that $\alpha+\beta=1, \alpha \beta=-1$ and $\alpha>0>\beta$.
+
+Since $\beta<0, \beta$ is not a solution.
+
+Now, if $x=\alpha$, then
+
+$$
+\begin{aligned}
+\left(\alpha-\frac{1}{\alpha}\right)^{1 / 2}+\left(1-\frac{1}{\alpha}\right)^{1 / 2} & =(\alpha+\beta)^{1 / 2}+(1+\beta)^{1 / 2} & & (\text { since } \alpha \beta--1) \\
+& =1^{1 / 2}+\left(\beta^{2}\right)^{1 / 2} & & \left(\text { since } \alpha+\beta=1 \text { and } \beta^{2}=\beta+1\right) \\
+& =1-\beta & & (\text { since } \beta<0) \\
+& =\alpha & & (\text { since } \alpha+\beta=1) .
+\end{aligned}
+$$
+
+So $x=\alpha$ is the unique solution to the equation.
+
+Solution 1 to Problem 3 - Chen He, Columbia International Collegiate, Hamilton, ON
+
+$$
+1+\frac{1}{3}+\ldots+\frac{1}{2 n-1}=\frac{1}{2}+\frac{1}{2}+\frac{1}{3}+\frac{1}{5}+\ldots \frac{1}{2 n-1}
+$$
+
+Since
+
+$$
+\frac{1}{3}>\frac{1}{4}, \frac{1}{5}>\frac{1}{6}, \ldots, \frac{1}{2 n-1}>\frac{1}{2 n}
+$$
+
+(1) gives
+
+$$
+1+\frac{1}{3}+\ldots+\frac{1}{2 n-1}>\frac{1}{2}+\frac{1}{2}+\frac{1}{4}+\frac{1}{6}+\ldots+\frac{1}{2 n}=\frac{1}{2}+\left(\frac{1}{2}+\frac{1}{4}+\frac{1}{6}+\ldots+\frac{1}{2 n}\right) .
+$$
+
+Since
+
+$$
+\frac{1}{2}>\frac{1}{4}, \frac{1}{2}>\frac{1}{6}, \frac{1}{2}>\frac{1}{8}, \ldots, \frac{1}{2}>\frac{1}{2 n}
+$$
+
+then
+
+$$
+\frac{n}{2}=\underbrace{\frac{1}{2}+\frac{1}{2}+\frac{1}{2}+\ldots+\frac{1}{2}}_{n}>\frac{1}{2}+\frac{1}{4}+\frac{1}{6}+\ldots+\frac{1}{2 n}
+$$
+
+So
+
+$$
+\frac{1}{2}>\frac{1}{n}\left(\frac{1}{2}+\frac{1}{4}+\frac{1}{6}+\ldots+\frac{1}{2 n}\right) .
+$$
+
+Then (1), (2) and (3) show
+
+$$
+\begin{aligned}
+1+\frac{1}{3}+\ldots+\frac{1}{2 n-1} & >\frac{1}{n}\left(\frac{1}{2}+\frac{1}{4}+\frac{1}{6}+\ldots+\frac{1}{2 n}\right)+\left(\frac{1}{2}+\frac{1}{4}+\frac{1}{6}+\ldots+\frac{1}{2 n}\right) \\
+& =\left(1+\frac{1}{n}\right)\left(\frac{1}{2}+\frac{1}{4}+\ldots+\frac{1}{2 n}\right) \\
+& =\frac{n+1}{n}\left(\frac{1}{2}+\frac{1}{4}+\ldots+\frac{1}{2 n}\right) .
+\end{aligned}
+$$
+
+Therefore $\frac{1}{n+1}\left(1+\frac{1}{3}+\ldots+\frac{1}{2 n-1}\right)>\frac{1}{n}\left(\frac{1}{2}+\frac{1}{4}+\ldots+\frac{1}{2 n}\right)$ for all $n \in N$ and $n \geq 2$.
+
+Solution 2 to Problem 3 - Yin Lei, Vincent Massey S.S., Windsor, ON
+
+Since $n \geq 2, n(n+1) \geq 0$. Therefore the given inequality is equivalent to
+
+$$
+n\left(1+\frac{1}{3}+\ldots+\frac{1}{2 n-1}\right) \geq(n+1)\left(\frac{1}{2}+\frac{1}{4}+\ldots+\frac{1}{2 n}\right) .
+$$
+
+We shall use mathematical induction to prove this.
+
+For $n=2$, obviously $\frac{1}{3}\left(1+\frac{1}{3}\right)=\frac{4}{9}>\frac{1}{2}\left(\frac{1}{2}+\frac{1}{4}\right)=\frac{3}{8}$.
+
+Suppose that the inequality stands for $n=k$, i.e.
+
+$$
+k\left(1+\frac{1}{3}+\ldots+\frac{1}{2 k-1}\right)>(k+1)\left(\frac{1}{2}+\frac{1}{4}+\ldots+\frac{1}{2 k}\right) .
+$$
+
+Now we have to prove it for $n=k+1$.
+
+We know
+
+$$
+\begin{aligned}
+& \left(1+\frac{1}{3}+\ldots+\frac{1}{2 k-1}\right)-\left(\frac{1}{2}+\frac{1}{4}+\ldots+\frac{1}{2 k}\right) \\
+& =\left(1-\frac{1}{2}\right)+\left(\frac{1}{3}-\frac{1}{4}\right)+\left(\frac{1}{5}-\frac{1}{6}\right)+\ldots+\left(\frac{1}{2 k-1}-\frac{1}{2 k}\right) \\
+& =\frac{1}{1 \times 2}+\frac{1}{3 \times 4}+\frac{1}{5 \times 6}+\ldots+\frac{1}{(2 k-1)(2 k)} .
+\end{aligned}
+$$
+
+Since
+
+$$
+1 \times 2<3 \times 4<5 \times 6<\ldots<(2 k-1)(2 k)<(2 k+1)(2 k+2)
+$$
+
+then
+
+$$
+\frac{1}{1 \times 2}+\frac{1}{3 \times 4}+\ldots+\frac{1}{(2 k-1)(2 k)}>\frac{k}{(2 k+1)(2 k+2)}
+$$
+
+hence
+
+$$
+1+\frac{1}{3}+\ldots+\frac{1}{2 k-1}>\frac{1}{2}+\frac{1}{4}+\ldots+\frac{1}{2 k}+\frac{k}{(2 k+1)(2 k+2)}
+$$
+
+Also
+
+$$
+\frac{k+1}{2 k+1}-\frac{k+2}{2 k+2}=\frac{2 k^{2}+2 k+2 k+2-2 k^{2}-4 k-k-2}{(2 k+1)(2 k+2)}=-\frac{k}{(2 k+1)(2 k+2)}
+$$
+
+therefore
+
+$$
+\frac{k+1}{2 k+1}=\frac{k+2}{2 k+2}-\frac{k}{(2 k+1)(2 k+2)}
+$$
+
+Adding 1, 2 and 3 :
+
+$k\left(1+\frac{1}{3}+\ldots+\frac{1}{2 k-1}\right)+\left(1+\frac{1}{3}+\ldots+\frac{1}{2 k-1}\right)+\frac{k+1}{2 k+1}$
+
+$>(k+1)\left(\frac{1}{2}+\frac{1}{4}+\ldots+\frac{1}{2 k}\right)+\left(\frac{1}{2}+\frac{1}{4}+\ldots+\frac{1}{2 k}\right)+\frac{k}{(2 k+1)(2 k+2)}+\frac{k+2}{2 k+2}-\frac{k}{(2 k+1)(2 k+2)}$
+
+Rearrange both sides to get
+
+$$
+(k+1)\left(1+\frac{1}{3}+\ldots+\frac{1}{2 k+1}\right)>(k+2)\left(\frac{1}{2}+\frac{1}{4}+\ldots+\frac{1}{2 k+2}\right) .
+$$
+
+Proving the induction.
+
+Solution 1 to Problem 4 - Keon Choi, A.Y. Jackson S.S., North York, ON
+
+Suppose $H$ is the foot of the perpendicular line from $A$ to $B C$; construct equilateral $\triangle A B G$, with $C$ on $B G$. I will prove that if $F$ is the point where $A H$ meets $B D$, then $\angle F C B=70^{\circ}$. (Because that means $A H$, and the given lines $B D$ and $C E$ meet at one point and that proves the question.) Suppose $B D$ extended meets $A G$ at $I$.
+
+
+
+Now $B F=G F$ and $\angle F B G=\angle F G B=40^{\circ}$ so that $\angle I G F=20^{\circ}$. Also $\angle I F G=\angle F B G+\angle F G B=$ $80^{\circ}$, so that
+
+$$
+\begin{aligned}
+\angle F I G & =180^{\circ}-\angle I F G-\angle I G F \\
+& =180^{\circ}-80^{\circ}-20^{\circ} \\
+& =80^{\circ} .
+\end{aligned}
+$$
+
+Therefore $\triangle G I F$ is an isoceles triangle, so
+
+$$
+G I=G F=B F
+$$
+
+But $\triangle B G I$ and $\triangle A B C$ are congruent, since $B G=A B, \angle G B I=\angle B A C, \angle B G I=\angle A B C$.
+
+Therefore
+
+$$
+G I=B C .
+$$
+
+From (1) and (2) we get
+
+$$
+B C=B F \text {. }
+$$
+
+So in $\triangle B C F$,
+
+$$
+\angle B C F=\frac{180^{\circ}-\angle F B C}{2}=\frac{180^{\circ}-40^{\circ}}{2}=70^{\circ} .
+$$
+
+Thus $\angle F C B=70^{\circ}$ and that proves that the given lines $C E$ and $B D$ and the perpendicular line $A H$ meet at one point.
+
+Solution 2 to Problem 4 - Adrian Birka, Lakeshore Catholic H.S., Port Colborne, ON
+
+First we prove the following lemma:
+
+In $\triangle A B C, A A^{\prime}, B B^{\prime}, C C^{\prime}$ intersect if-f
+
+$$
+\frac{\sin \alpha_{1}}{\sin \alpha_{2}} \cdot \frac{\sin \beta_{1}}{\sin \beta_{2}} \cdot \frac{\sin \gamma_{1}}{\sin \gamma_{2}}=1
+$$
+
+where $\alpha_{1}, \alpha_{2}, \beta_{1}, \beta_{2}, \gamma_{1}, \gamma_{2}$ are as shown in the diagram just below.
+
+(Editor: This is a known variant of Ceva's Theorem.)
+
+
+
+Proof: Let $\angle B B^{\prime} C=x$, then $\angle B B^{\prime} A=180^{\circ}-x$. Using the sine law in $\triangle B B^{\prime} C$ yields
+
+$$
+\frac{b_{2}}{\sin \beta_{2}}=\frac{a}{\sin x} .
+$$
+
+Similarly using the sine law in $\triangle B B^{\prime} A$ yields
+
+$$
+\frac{b_{1}}{\sin \beta_{1}}=\frac{c}{\sin \left(180^{\circ}-x\right)}=\frac{c}{\sin x} .
+$$
+
+Hence,
+
+$$
+b_{1}: b_{2}=\frac{c \sin \beta_{1}}{a \sin \beta_{2}}
+$$
+
+(from (1),(2)). (Editor: Do you recognize this when $\beta_{1}=\beta_{2}$ ?)
+
+Similarly,
+
+$$
+a_{1}: a_{2}=\frac{b \sin \alpha_{1}}{c \sin \alpha_{2}}, \quad c_{1}: c_{2}=\frac{a \sin \gamma_{1}}{b \sin \gamma_{2}}
+$$
+
+By Ceva's theorem, the necessary and sufficient condition for $A A^{\prime}, B B^{\prime}, C C^{\prime}$ to intersect is: $\left(a_{1}: a_{2}\right) \cdot\left(b_{1}: b_{2}\right) \cdot\left(c_{1}: c_{2}\right)=1$. Using (3), (4) on this yields:
+
+$$
+\frac{b}{c} \cdot \frac{\sin \alpha_{1}}{\sin \alpha_{2}} \cdot \frac{a}{b} \cdot \frac{\sin \gamma_{1}}{\sin \gamma_{2}} \cdot \frac{c}{a} \cdot \frac{\sin \beta_{1}}{\sin \beta_{2}}=1
+$$
+
+$$
+\frac{\sin \alpha_{1}}{\sin \alpha_{2}} \cdot \frac{\sin \beta_{1}}{\sin \beta_{2}} \cdot \frac{\sin \gamma_{1}}{\sin \gamma_{2}}=1
+$$
+
+This is just what we needed to show, therefore the lemma is proved.
+
+Now, in our original question, give $\angle B A C=40^{\circ}, \angle A B C=60^{\circ}$. It follows that $\angle A C B=80^{\circ}$.
+
+Since $\angle C B D=40^{\circ}, \angle A B D=\angle A B C-\angle D B C=20^{\circ}$. Similarly, $\angle E C A=20^{\circ}$.
+
+
+
+Now let us show that $\angle F A D=10^{\circ}$. Suppose otherwise. Let $F^{\prime}$ be such that $F, F^{\prime}$ are in the same side of $A C$ and $\angle D A F^{\prime}=10^{\circ}$. Then $\angle B A F^{\prime}=\angle B A C-\angle D A F^{\prime}=30^{\circ}$.
+
+Thus
+
+$$
+\begin{aligned}
+\frac{\sin \angle A B D}{\sin \angle D B C} \cdot \frac{\sin \angle B C E}{\sin \angle E C A} \cdot \frac{\sin \angle C A F^{\prime}}{\sin \angle F^{\prime} A B} & =\frac{\sin 20^{\circ}}{\sin 40^{\circ}} \cdot \frac{\sin 70^{\circ}}{\sin 10^{\circ}} \cdot \frac{\sin 10^{\circ}}{\sin 30^{\circ}} \\
+& =\frac{\sin 20^{\circ}}{2 \sin 20^{\circ} \cos 20^{\circ}} \cdot \frac{\cos 20^{\circ}}{\sin 30^{\circ}} \\
+& =\frac{1}{2 \sin 30^{\circ}}=1 .
+\end{aligned}
+$$
+
+By the lemma above, $A F^{\prime}$ passes through $C E \cap B D=F$. Therefore $A F^{\prime}=A F$, and $\angle F A D=10^{\circ}$, contrary to assumption. Thus $\angle F A D$ must be $10^{\circ}$. Now let $A F \cap B C=K$. Since $\angle K A C=$ $10^{\circ}, \angle K C A=80^{\circ}$, it follows that $\angle A K C=90^{\circ}$. Therefore $A K \perp B C \Rightarrow A F \perp B C$ as needed.
+
+Solution to Problem 5 - Adrian Chan, Upper Canada College, Toronto, ON
+
+Let us first prove by induction that $\frac{a_{n}^{2}+a_{n+1}^{2}}{a_{n} \cdot a_{n+1}+1}=m^{2}$ for all $n \geq 0$.
+
+Proof:
+
+Base Case $(n=0): \frac{a_{0}^{2}+a_{1}^{2}}{a_{0} \cdot a_{1}+1}=\frac{0+m^{2}}{0+1}=m^{2}$.
+
+Now, let us assume that it is true for $n=k, k \geq 0$. Then,
+
+$$
+\begin{aligned}
+\frac{a_{k}^{2}+a_{k+1}^{2}}{a_{k} \cdot a_{k+1}+1} & =m^{2} \\
+a_{k}^{2}+a_{k+1}^{2} & =m^{2} \cdot a_{k} \cdot a_{k+1}+m^{2} \\
+a_{k+1}^{2}+m^{4} a_{k+1}^{2}-2 m^{2} \cdot a_{k} \cdot a_{k+1}+a_{k}^{2} & =m^{2}+m^{4} a_{k+1}^{2}-m^{2} \cdot a_{k} \cdot a_{k+1} \\
+a_{k+1}^{2}+\left(m^{2} a_{k+1}-a_{k}\right)^{2} & =m^{2}+m^{2} a_{k+1}\left(m^{2} a_{k+1}-a_{k}\right) \\
+a_{k+1}^{2}+a_{k+2}^{2} & =m^{2}+m^{2} \cdot a_{k+1} \cdot a_{k+2} .
+\end{aligned}
+$$
+
+So $\frac{a_{k+1}^{2}+a_{k+2}^{2}}{a_{k+1} \cdot a_{k+2}+1}=m^{2}$,
+
+proving the induction. Hence $\left(a_{n}, a_{n+1}\right)$ is a solution to $\frac{a^{2}+b^{2}}{a b+1}=m^{2}$ for all $n \geq 0$.
+
+Now, consider the equation $\frac{a^{2}+b^{2}}{a b+1}=m^{2}$ and suppose $(a, b)=(x, y)$ is a solution with $0 \leq x \leq y$. Then
+
+$$
+\frac{x^{2}+y^{2}}{x y+1}=m^{2}
+$$
+
+If $x=0$ then it is easily seen that $y=m$, so $(x, y)=\left(a_{0}, a_{1}\right)$. Since we are given $x \geq 0$, suppose now that $x>0$.
+
+Let us show that $y \leq m^{2} x$.
+
+Proof by contradiction: Assume that $y>m^{2} x$. Then $y=m^{2} x+k$ where $k \geq 1$.
+
+Substituting into (1) we get
+
+$$
+\begin{aligned}
+\frac{x^{2}+\left(m^{2} x+k\right)^{2}}{(x)\left(m^{2} x+k\right)+1} & =m^{2} \\
+x^{2}+m^{4} x^{2}+2 m^{2} x k+k^{2} & =m^{4} x^{2}+m^{2} k x+m^{2} \\
+\left(x^{2}+k^{2}\right)+m^{2}(k x-1) & =0 .
+\end{aligned}
+$$
+
+Now, $m^{2}(k x-1) \geq 0$ since $k x \geq 1$ and $x^{2}+k^{2} \geq x^{2}+1 \geq 1$ so $\left(x^{2}+k^{2}\right)+m^{2}(k x-1) \neq 0$.
+
+Thus we have a contradiction, so $y \leq m^{2} x$ if $x>0$.
+
+Now substitute $y=m^{2} x-x_{1}$, where $0 \leq x_{1} a_{2}$, then $u_{1}$ is a multiple of $u_{2}$ and if $a_{1} C$ distance to go, which is not enough for a big jump to clear the lava. Thus, he must do at least $n$ jumps in the safe interval, but that's possible only with all small jumps, and furthermore is impossible if the starting position is $C$. This gives him starting position 1 higher in the next safe interval, so sooner or later the flea is going to hit the lava.
+
+Flea's strategy: The flea just does one interval at a time. If the safe interval has at least $n A+C$ integers in it, the flea has distance $d>n A$ to go to the next lava when it starts. Repeatedly do big jumps until $d$ is between 1 and $C \bmod A$, then small jumps until the remaining distance is between 1 and $C$, then a final big jump. This works as long as the first part does. However, we get at least $n$ big jumps since floor $((d-1) / A)$ can never go down two from a big jump (or we'd be done doing big jumps), so we get $n$ big jumps, and thus we are good if $d \bmod A$ is in any of $[1, C],[C+1,2 C], \ldots[n C+1,(n+1) C]$, but that's everything.
+
+Let $C=B-A$. We shall write our intervals of lava in the form $\left(L_{i}, R_{i}\right]=\left\{L_{i}+1, L_{i}+2, \ldots, R_{i}\right\}$, where $R_{i}=L_{i}+A$ and $R_{i-1}
1990
+
+## PROBLEM 1
+
+A competition involving $n \geq 2$ players was held over $k$ days. On each day, the players received scores of $1,2,3, \ldots, n$ points with no two players receiving the same score. At the end of the $k$ days, it was found that each player had exactly 26 points in total. Determine all pairs $(n, k)$ for which this is possible.
+
+## Problem 2
+
+A set of $\frac{1}{2} n(n+1)$ distinct numbers is arranged at random in a triangular array:
+
+
+Let $M_{k}$ be the largest number in the $k$-th row from the top. Find the probability that
+
+$$
+M_{1}
SOLUTIONS
+
+Most of the solutions to the problems of the 1999 CMO presented below are taken from students' papers. Some minor editing has been done - unnecessary steps have been eliminated and some wording has been changed to make the proofs clearer. But for the most part, the proofs are as submitted.
+
+## Solution to Problem 1 - Adrian Chan, Upper Canada College, Toronto, ON
+
+Rearranging the equation we get $4 x^{2}+51=40[x]$. It is known that $x \geq[x]>x-1$, so
+
+$$
+\begin{aligned}
+4 x^{2}+51=40[x] & >40(x-1) \\
+4 x^{2}-40 x+91 & >0 \\
+(2 x-13)(2 x-7) & >0
+\end{aligned}
+$$
+
+Hence $x>13 / 2$ or $x<7 / 2$. Also,
+
+$$
+\begin{aligned}
+4 x^{2}+51=40[x] & \leq 40 x \\
+4 x^{2}-40 x+51 & \leq 0 \\
+(2 x-17)(2 x-3) & \leq 0
+\end{aligned}
+$$
+
+Hence $3 / 2 \leq x \leq 17 / 2$. Combining these inequalities gives $3 / 2 \leq x<7 / 2$ or $13 / 2
Chair: Luis Goddyn, Simon Fraser University, goddyn@math.sfu.ca
+
+The Year 2000 Canadian Mathematics Olympiad was written on Wednesday April 2, by 98 high school students across Canada. A correct and well presented solution to any of the five questions was awarded seven points. This year's exam was a somewhat harder than usual, with the mean score being 8.37 out of 35 . The top few scores were: 30, 28, 27, 22, 20, 20, 20. The first, second and third prizes are awarded to: Daniel Brox (Sentinel Secondary BC), David Arthur (Upper Canada College ON), and David Pritchard (Woburn Collegiate Institute ON).
+
+1. At 12:00 noon, Anne, Beth and Carmen begin running laps around a circular track of length three hundred meters, all starting from the same point on the track. Each jogger maintains a constant speed in one of the two possible directions for an indefinite period of time. Show that if Anne's speed is different from the other two speeds, then at some later time Anne will be at least one hundred meters from each of the other runners. (Here, distance is measured along the shorter of the two arcs separating two runners.)
+
+Comment: We were surprised by the difficulty of this question, having awarded an average grade of 1.43 out of 7 . We present two solutions; only the first appeared among the graded papers.
+
+Solution 1: By rotating the frame of reference we may assume that Anne has speed zero, that Beth runs at least as fast as Carmen, and that Carmen's speed is positive. If Beth is no more than twice as fast as Carmen, then both are at least 100 meters from Anne when Carmen has run 100 meters. If Beth runs more that twice as fast as Carmen, then Beth runs a stretch of more than 200 meters during the time Carmen runs between 100 and 200 meters. Some part of this stretch lies more than 100 meters from Anne, at which time both Beth and Carmen are at least (in fact, more than) 100 meters away from Anne.
+
+Solution 2: By rotating the frame of reference we may assume Anne's speed to equal zero, and that the other two runners have non-zero speed. We may assume that Beth is running at least as fast as Carmen. Suppose that it takes $t$ seconds for Beth to run 200 meters. Then at all times in the infinite set $T=\{t, 2 t, 4 t, 8 t, \ldots\}$, Beth is exactly 100 meters from Anne. At time $t$, Carmen has traveled exactly $d$ meters where $0
Life Financial
+
+
+that can cover this unit square and a diagonal square is a $4 \times 4$ square tile.
+
+
+
+Continuing this construction, we see that the side lengths of the square tiles we encounter will be $1,2,4$, and so on, up to $2^{k}$ for some nonnegative integer $k$. Therefore, $n$, the height of the $n$-staircase, is equal to $1+2+4+\cdots+2^{k}=2^{k+1}-1$. Alternatively, $n=2^{k}-1$ for some positive integer $k$. Let $p(k)=2^{k}-1$.
+
+Conversely, we can tile a $p(k)$-staircase with $p(k)$ square tiles recursively as follows: We have that $p(1)=1$, and we can tile a 1 -staircase with 1 square tile. Assume that we can tile a $p(k)$-staircase with $p(k)$ square tiles for some positive integer $k$.
+
+Consider a $p(k+1)$-staircase. Place a $2^{k} \times 2^{k}$ square tile in the bottom left corner. Note that this square tile covers a digaonal square. Then $p(k+1)-2^{k}=$ $2^{k+1}-1-2^{k}=2^{k}-1=p(k)$, so we are left with two $p(k)$-staircases.
+
+
+
+Furthermore, these two $p(k)$-staircases can be tiled with $2 p(k)$ square tiles, which means we use $2 p(k)+1=p(k+1)$ square tiles.
+
+Therefore, $f(n)=n$ if and only if $n=2^{k}-1=p(k)$ for some positive integer $k$. In other words, the binary representation of $n$ consists of all $1 \mathrm{~s}$, with no $0 \mathrm{~s}$.
+
+(b) Let $n$ be a positive integer such that $f(n)=n+1$, and consider a minimal tiling of an $n$-staircase. Since there are $n$ diagonal squares, every square tile except one covers a diagonal square. We claim that the square tile that covers the bottom-left unit square must be the square tile that does not cover a diagonal square.
+
+If $n$ is even, then this fact is obvious, because the square tile that covers the bottom-left unit square cannot cover any diagonal square, so assume that $n$ is odd. Let $n=2 m+1$. We may assume that $n>1$, so $m \geq 1$. Suppose that the square tile covering the bottom-left unit square also covers a diagonal square. Then the side length of this square tile must be $m+1$. After this $(m+1) \times(m+1)$ square tile has been placed, we are left with two $m$-staircases.
+
+
+
+Hence, $f(n)=2 f(m)+1$. But $2 f(m)+1$ is odd, and $n+1=2 m+2$ is even, so $f(n)$ cannot be equal to $n+1$, contradiction. Therefore, the square tile that covers the bottom-left unit square is the square tile that does not cover a diagonal square.
+
+Let $t$ be the side length of the square tile covering the bottom-left unit square. Then every other square tile must cover a diagonal square, so by the same construction as in part (a), $n=1+2+4+\cdots+2^{k-1}+t=2^{k}+t-1$ for some positive integer $k$. Furthermore, the top $p(k)=2^{k}-1$ rows of the $n$-staircase must be tiled the same way as the minimal tiling of a $p(k)$-staircase. Therefore, the horizontal line between rows $p(k)$ and $p(k)+1$ does not pass through any square tiles. Let us call such a line a fault line. Similarly, the vertical line between columns $t$ and $t+1$ is also a fault line. These two fault lines partition two $p(k)$-staircases.
+
+
+
+If these two $p(k)$-staircases do not overlap, then $t=p(k)$, so $n=2 p(k)$. For example, the minimal tiling for $n=2 p(2)=6$ is shown below.
+
+
+
+Hence, assume that the two $p(k)$-staircases do overlap. The intersection of the two $p(k)$-staircases is a $[p(k)-t]$-staircase. Since this $[p(k)-t]$-staircase is tiled the same way as the top $p(k)-t$ rows of a minimal tiling of a $p(k)$-staircase, $p(k)-t=p(l)$ for some positive integer $l
https://cms.math.ca/Competitions/Sponsors/
+
+1. Let $S$ be a set of $n \geq 3$ positive real numbers. Show that the largest possible number of distinct integer powers of three that can be written as the sum of three distinct elements of $S$ is $n-2$.
+
+Solution: We will show by induction that for all $n \geq 3$, it holds that at most $n-2$ powers of three are sums of three distinct elements of $S$ for any set $S$ of positive real numbers with $|S|=n$. This is trivially true when $n=3$. Let $n \geq 4$ and consider the largest element $x \in S$. The sum of $x$ and any two other elements of $S$ is strictly between $x$ and $3 x$. Therefore $x$ can be used as a summand for at most one power of three. By the induction hypothesis, at most $n-3$ powers of three are sums of three distinct elements of $S \backslash\{x\}$. This completes the induction.
+
+Even if it was not asked to prove, we will now show that the optimal answer $n-2$ is reached. Observe that the set $S=\left\{1,2,3^{2}-3,3^{3}-3, \ldots, 3^{n}-3\right\}$ is such that $3^{2}, 3^{3}, \ldots, 3^{n}$ can be expressed as sums of three distinct elements of $S$. This makes use of the fact that each term of the form $3^{k}-3$ can be used in exactly one sum of three terms equal to $3^{k}$.
+
+## A competition of the Canadian Mathematical Society and
+
+ supported by the Actuarial Profession.
+
+## Expertise. Insight. Solutions.
+
+2. A circle is inscribed in a rhombus $A B C D$. Points $P$ and $Q$ vary on line segments $\overline{A B}$ and $\overline{A D}$, respectively, so that $\overline{P Q}$ is tangent to the circle. Show that for all such line segments $\overline{P Q}$, the area of triangle $C P Q$ is constant.
+
+
+
+## Solution.
+
+Let the circle be tangent to $\overline{P Q}, \overline{A B}, \overline{A D}$ at $T, U$, and $V$, respectively. Let $p=P T=P U$ and $q=Q T=Q V$. Let $a=A U=A V$ and $b=B U=D V$. Then the side length of the rhombus is $a+b$.
+
+
+
+Let $\theta=\angle B A D$, so $\angle A B C=\angle A D C=180^{\circ}-\theta$. Then (using the notation [XYZ] for the area of a triangle of vertices $X, Y, Z$ )
+
+$$
+\begin{aligned}
+& {[A P Q]=\frac{1}{2} \cdot A P \cdot A Q \cdot \sin \theta=\frac{1}{2}(a-p)(a-q) \sin \theta} \\
+& {[B C P]=\frac{1}{2} \cdot B P \cdot B C \cdot \sin \left(180^{\circ}-\theta\right)=\frac{1}{2}(b+p)(a+b) \sin \theta} \\
+& {[C D Q]=\frac{1}{2} \cdot D Q \cdot C D \cdot \sin \left(180^{\circ}-\theta\right)=\frac{1}{2}(b+q)(a+b) \sin \theta}
+\end{aligned}
+$$
+
+SO
+
+$$
+\begin{aligned}
+{[C P Q] } & =[A B C D]-[A P Q]-[B C P]-[C D Q] \\
+& =(a+b)^{2} \sin \theta-\frac{1}{2}(a-p)(a-q) \sin \theta-\frac{1}{2}(b+p)(a+b) \sin \theta-\frac{1}{2}(b+q)(a+b) \sin \theta \\
+& =\frac{1}{2}\left(a^{2}+2 a b-b p-b q-p q\right) \sin \theta
+\end{aligned}
+$$
+
+Let $O$ be the center of the circle, and let $r$ be the radius of the circle. Let $x=\angle T O P=\angle U O P$ and $y=\angle T O Q=\angle V O Q$. Then $\tan x=\frac{p}{r}$ and $\tan y=\frac{q}{r}$.
+
+
+
+Note that $\angle U O V=2 x+2 y$, so $\angle A O U=x+y$. Also, $\angle A O B=90^{\circ}$, so $\angle O B U=x+y$. Therefore,
+
+$$
+\tan (x+y)=\frac{a}{r}=\frac{r}{b}
+$$
+
+so $r^{2}=a b$. But
+
+$$
+\frac{r}{b}=\tan (x+y)=\frac{\tan x+\tan y}{1-\tan x \tan y}=\frac{\frac{p}{r}+\frac{q}{r}}{1-\frac{p}{r} \cdot \frac{q}{r}}=\frac{r(p+q)}{r^{2}-p q}=\frac{r(p+q)}{a b-p q} .
+$$
+
+Hence, $a b-p q=b p+b q$, so $b p+b q+p q=a b$. Therefore,
+
+$$
+[C P Q]=\frac{1}{2}\left(a^{2}+2 a b-b p-b q-p q\right) \sin \theta=\frac{1}{2}\left(a^{2}+a b\right) \sin \theta
+$$
+
+which is constant.
+
+Alternate Solution: Let $O$ be the center of the circle and $r$ its radius. Then $[C P Q]=[C D Q P B]-$ $[C D Q]-[C B P]$, where $[\ldots]$ denotes area of the polygon with given vertices. Note that $[C D Q P B]$ is half $r$ times the perimeter of $C D Q P B$. Note that the heights of $C D Q$ and $C B P$ are $2 r$ so $[C D Q]=r \cdot D Q$ and $[C B P]=r \cdot P B$. Using the fact that $Q T=Q V$ and $P U=P T$, it now follows that $[C P Q]=[O V D C B U]-[C D V]-[C B U]$, which is independent of $P$ and $Q$.
+
+3. A purse contains a finite number of coins, each with distinct positive integer values. Is it possible that there are exactly 2020 ways to use coins from the purse to make the value 2020 ?
+
+Solution: It is possible.
+
+Consider a coin purse with coins of values 2,4,8,2014,2016,2018, 2020 and every odd number between 503 and 1517. Call such a coin big if its value is between 503 and 1517. Call a coin small if its value is 2,4 or 8 and huge if its value is 2014,2016,2018 or 2020. Suppose some subset of these coins contains no huge coins and sums to 2020. If it contains at least four big coins, then its value must be at least $503+505+507+509>2020$. Furthermore since all of the small coins are even in value, if the subset contains exactly one or three big coins, then its value must be odd. Thus the subset must contain exactly two big coins. The eight possible subsets of the small coins have values $0,2,4,6,8,10,12,14$. Therefore the ways to make the value 2020 using no huge coins correspond to the pairs of big coins with sums 2006,2008,2010,2012,2014,2016,2018 and 2020. The numbers of such pairs are $250,251,251,252,252,253,253,254$, respectively. Thus there are exactly 2016 subsets of this coin purse with value 2020 using no huge coins. There are exactly four ways to make a value of 2020 using huge coins; these are $\{2020\},\{2,2018\},\{4,2016\}$ and $\{2,4,2014\}$. Thus there are exactly 2020 ways to make the value 2020 .
+
+Alternate construction: Take the coins $1,2, \ldots, 11,1954,1955, \ldots, 2019$. The only way to get 2020 is a non-empty subset of $1, \ldots, 11$ and a single large coin. There are 2047 non-empty such subsets of sums between 1 and 66 . Thus they each correspond to a unique large coin making 2020, so we have 2047 ways. Thus we only need to remove some large coins, so that we remove exactly 27 small sums. This can be done, for example, by removing coins $2020-n$ for $n=1,5,6,7,8,9$, as these correspond to $1+3+4+5+6+8=27$ partitions into distinct numbers that are at most 11 .
+
+4. Let $S=\{1,4,8,9,16, \ldots\}$ be the set of perfect powers of integers, i.e. numbers of the form $n^{k}$ where $n, k$ are positive integers and $k \geq 2$. Write $S=\left\{a_{1}, a_{2}, a_{3} \ldots\right\}$ with terms in increasing order, so that $a_{1}